Prove the Sum Rule for Limits

  • Thread starter Tsunoyukami
  • Start date
  • Tags
    Limits Sum
In summary, the Sum Rule for Limits is a fundamental concept in calculus that states that the limit of the sum of two functions is equal to the sum of their individual limits. It is used to simplify and solve more complex limits and is also utilized in finding derivatives and integrals. The Sum Rule for Limits can be proven using the formal definition of a limit and is valid when both functions have finite limits at the same point. However, there are exceptions to this rule when either of the individual limits is infinite or does not exist.
  • #1
Tsunoyukami
215
11
Prove the Sum Rule for Limits

$$\lim_{x\to a} [f(x) + g(x)] = \lim_{x \to a} f(x) + \lim_{x \to a} g(x) = L + M$$

Proof

Assume the following:
$$\lim_{x \to a} f(x) = L, \space\lim_{x \to a} g(x) = M$$
Then, by definition
##\forall \epsilon_1 > 0, \exists \delta_1 > 0## such that ##0<|x-a|<\delta_1 \implies |f(x)-L|<\epsilon_1##
and
##\forall \epsilon_2 >0, \exists \delta_2 > 0## such that ##0<|x-a|<\delta_2 \implies |g(x)-M|<\epsilon_2##

Choose ##\delta = \min(\delta_1, \delta_2)##.

Then ##0<|x-a|<\delta \implies |f(x)-L|<\epsilon_1## and ##|g(x)-M|<\epsilon_2##.

Notice ##|[f(x) + g(x)] - [L+M]| = |[f(x)-L] + [g(x)-M]| \leq |f(x)-L| + |g(x)-M|## by the triangle inequality. Then ##|[f(x) + g(x)] - [L+M]| \leq |f(x)-L| + |g(x)-M| < \epsilon_1 + \epsilon_2 = \epsilon##.

Then ##|[f(x) + g(x)] - [L+M]| < \epsilon##. Therefore we conclude

$$\lim_{x\to a} [f(x) + g(x)] = \lim_{x \to a} f(x) + \lim_{x \to a} g(x) = L + M$$



I was comparing my proof (the one above) to the proof offered on page 93 of James Stewart's Calculus (6e) and noticed that instead of ##\epsilon_1## and ##\epsilon_2## he uses ##\frac{\epsilon}{2}## for each of these so that their sum is ##\epsilon##.

Is there anything incorrect with my method? I figure that my approach with two different possible values of ##\epsilon_i## for each of the limits is just a more generalized way of saying the same thing - but the other proofs I've looked at online all use ##\frac{\epsilon}{2}## - is there any reason for this?

Thanks in advance!
 
Physics news on Phys.org
  • #2
Tsunoyukami said:
Prove the Sum Rule for Limits

$$\lim_{x\to a} [f(x) + g(x)] = \lim_{x \to a} f(x) + \lim_{x \to a} g(x) = L + M$$

Proof

Assume the following:
$$\lim_{x \to a} f(x) = L, \space\lim_{x \to a} g(x) = M$$
Then, by definition
##\forall \epsilon_1 > 0, \exists \delta_1 > 0## such that ##0<|x-a|<\delta_1 \implies |f(x)-L|<\epsilon_1##
and
##\forall \epsilon_2 >0, \exists \delta_2 > 0## such that ##0<|x-a|<\delta_2 \implies |g(x)-M|<\epsilon_2##

Choose ##\delta = \min(\delta_1, \delta_2)##.

Then ##0<|x-a|<\delta \implies |f(x)-L|<\epsilon_1## and ##|g(x)-M|<\epsilon_2##.

Notice ##|[f(x) + g(x)] - [L+M]| = |[f(x)-L] + [g(x)-M]| \leq |f(x)-L| + |g(x)-M|## by the triangle inequality. Then ##|[f(x) + g(x)] - [L+M]| \leq |f(x)-L| + |g(x)-M| < \epsilon_1 + \epsilon_2 = \epsilon##.

Then ##|[f(x) + g(x)] - [L+M]| < \epsilon##. Therefore we conclude

$$\lim_{x\to a} [f(x) + g(x)] = \lim_{x \to a} f(x) + \lim_{x \to a} g(x) = L + M$$
I was comparing my proof (the one above) to the proof offered on page 93 of James Stewart's Calculus (6e) and noticed that instead of ##\epsilon_1## and ##\epsilon_2## he uses ##\frac{\epsilon}{2}## for each of these so that their sum is ##\epsilon##.

Is there anything incorrect with my method? I figure that my approach with two different possible values of ##\epsilon_i## for each of the limits is just a more generalized way of saying the same thing - but the other proofs I've looked at online all use ##\frac{\epsilon}{2}## - is there any reason for this?

Thanks in advance!

If you had started with a statement of what you want to prove, you would have written:

To prove: Given ##\epsilon > 0##, show there is ##\delta > 0## such that if ##0<|x-a|< \delta## then ##|(f(x)+g(x)) - (L+M)| < \epsilon##. And that is why you would pick the intermediate values in your argument so it all comes out less than ##\epsilon## at the end.

Your proof should start with "Suppose ##\epsilon > 0##" Then give your argument.
 
  • #3
Oh, I see so it would be something like this:

Proof
Let ##\epsilon>0## be given. We must find ##\delta > 0## such that ##0<|x-a|<\delta \implies |f(x)+g(x) -(L+M)|<\epsilon##. By applying the triangle inequality we can write ##|f(x)+g(x) - (L+M)| \leq |f(x)-L| + |g(x)-M|##.

I will pause the proof here momentarily. The proofs I have seen have continued by letting each of these terms be less than ##\frac{\epsilon}{2}## - my question is whether or not I could let one of them be less than ##\frac{\epsilon}{3}## and the other be less than ##\frac{2\epsilon}{3}## - the sum of these terms is still ##\epsilon## so this should still be valid correct? I will attempt to complete the proof using these values.

Getting back to the main body of the proof:

Let ##|f(x) - L|<\frac{\epsilon}{3}## and ##|g(x)-M|<\frac{2\epsilon}{3}##. Since ##\epsilon>0## it follows that both ##\frac{\epsilon}{3}>0## and ##\frac{2\epsilon}{3}>0##. Since both the limit of ##f(x)## and ##g(x)## exist as ##x## approaches ##a## there exist ##\delta_1## and ##\delta_2## satisfying the following conditions:

$$0<|x-a|<\delta_1 \implies |f(x)-L|<\frac{\epsilon}{3}$$
$$0<|x-a|<\delta_2 \implies |g(x)-M|<\frac{2\epsilon}{3}$$

Choose ##\delta=\min(\delta_1, \delta_2)##. If ##0<|x-a|<\delta## then ##0<|x-a|<\delta_1## and ##0<|x-a|<\delta_2## so ##|f(x)-L|<\frac{\epsilon}{3}## and ##|g(x)-M|<\frac{2\epsilon}{3}##.

Then ##|f(x)+g(x) - (L+M)| \leq |f(x)-L| + |g(x)-M| < \frac{\epsilon}{3} + \frac{2\epsilon}{3} = \epsilon##.

This completes the proof.
What was bothering me was that every proof I saw relied upon splitting the two terms equally into ##\frac{\epsilon}{2}## when I figured that it should be possible to split ##\epsilon## into two uneven terms.

Does the above constitute a valid proof?
 
  • #4
Yes. It doesn't matter how you break it up as long as the total parts add to less than ##\epsilon##.
 
  • Like
Likes 1 person

What is the Sum Rule for Limits?

The Sum Rule for Limits is a fundamental concept in calculus that states that the limit of the sum of two functions is equal to the sum of their individual limits.

How is the Sum Rule for Limits used in calculus?

The Sum Rule for Limits is used to simplify and solve more complex limits by breaking them down into smaller, easier to solve limits. It is also used in finding derivatives and integrals of functions.

Can you prove the Sum Rule for Limits?

Yes, the Sum Rule for Limits can be proven using the formal definition of a limit and the properties of limits, such as the limit of a sum is equal to the sum of the limits.

What are the conditions for the Sum Rule for Limits to be valid?

The Sum Rule for Limits is valid when both functions have finite limits at the same point, and the limit of the sum is equal to the sum of the limits. In other words, the two functions must be continuous at the point in question.

Are there any exceptions to the Sum Rule for Limits?

Yes, the Sum Rule for Limits does not hold if either of the individual limits is infinite or does not exist. In these cases, the limit of the sum may still exist, but it cannot be determined using the Sum Rule for Limits.

Similar threads

  • Calculus and Beyond Homework Help
Replies
5
Views
880
  • Calculus and Beyond Homework Help
Replies
32
Views
2K
  • Calculus and Beyond Homework Help
Replies
9
Views
2K
  • Calculus and Beyond Homework Help
Replies
2
Views
845
  • Calculus and Beyond Homework Help
Replies
16
Views
1K
  • Calculus and Beyond Homework Help
Replies
8
Views
1K
  • Calculus and Beyond Homework Help
Replies
13
Views
2K
  • Calculus and Beyond Homework Help
Replies
6
Views
1K
  • Calculus and Beyond Homework Help
Replies
16
Views
2K
  • Calculus and Beyond Homework Help
Replies
8
Views
2K
Back
Top